正在加载图片...
于是得到1-2的置信水平为1-a的置信区间为 ⅹ-Y-tn+-2)S。1+-,x-P+ta2(n1+n2-2./1,1 Vn,n, 其中Sn=/(n1-1 S;2+(n,-1)S2 n1+n2-2 这里a/2=0.025,n1=10,n2=20,n1+n2-2=28, 62(28)=2.048.x=500,y=496,So=1.1688. 故两总体均值差1-2的置信水平为0.95的置信区 间为(3.07,4.93)于是得到 μ1 2 − μ 的置信水平为 1− α 的置信区间为         − − + − + − + + − + 1 2 2 1 2 1 2 2 1 2 1 1 , ( 2) 1 1 ( 2) n n X Y t n n S n n X Y t a n n S a  这里 1 2 1 2 α 2 0.025, 10 , 20, 2 28, = = = + − = n n n n t 0.025 (28) 2.048. = 1.1688. ω x = 500, y = 496, s = 故两总体均值差 的置信水平为0.95 的置信区 间为 μ1 2 − μ (3.07, 4.93) . 其中 2 2 1 1 2 2 1 2 ( 1) ( 1) . 2 ω n S n S S n n − + − = + −
<<向上翻页向下翻页>>
©2008-现在 cucdc.com 高等教育资讯网 版权所有